PDA

Archiv verlassen und diese Seite im Standarddesign anzeigen : Gesunder Menschenverstand reicht nicht aus


Eyk van Bommel
23.07.09, 16:17
Quantenmessungen (http://www.pro-physik.de/Phy/leadArticle.do?laid=12030): Gesunder Menschenverstand reicht nicht aus

Was sollen uns (oder mir) diese Worte sagen?

A) Es gibt KEINE lokalen, verborgenen Variablen :confused:
B) Es kann, wenn überhaupt, nur nicht-lokale Variablen geben :confused:
C) Erklärungsmodelle die übrig bleiben: Spukhafte Fernwirkung oder VWT:confused:

Ich bin für Spukhafte Fernwirkung:D

Gruß
EVB

Uli
23.07.09, 16:28
...
Eine theoretische Überlegung von Simon Kochen und Ernst Specker aus dem Jahr 1967 zeigt jedoch, dass bei solchen Erklärungsversuchen mit verborgenen Variablen die Messungen kontextuell sein müssen. Das heißt, dass das Ergebnis einer Messung von anderen gleichzeitig durchgeführten Messungen abhängig ist. Interessanterweise sind die Messungen hierbei miteinander verträglich und stören sich nicht gegenseitig.
...
Mit diesem technologischen Vorsprung konnten die Forscher nun erstmals auch mit experimentellen Mitteln umfassend nachweisen, dass nichtkontextuelle Erklärungsmodelle mit verborgenen Variablen nicht mit den Experimenten vereinbar sind. Das Ergebnis ist dabei unabhängig vom verwendeten Quantenzustand, im Experiment wurde es an zehn verschiedenen Zuständen getestet.
...


Also - ehrlich gesagt - ich verstehe nur Bahnhof.
Was haben die da wie ausgeschlossen ?

SCR
23.07.09, 16:54
Es geht primär um den Ausschluß nicht-kontextueller (= Ergebnis einer Messung ist von anderen gleichzeitig durchgeführten Messungen unabhängig) Erklärungsmodelle
-> Erklärungsmodelle auf Basis verborgener Variablen müssen "wenn dann schon" kontextueller (= Ergebnis einer Messung ist von anderen gleichzeitig durchgeführten Messungen abhängig) Natur sein.

Hier wurde anscheinend das KS-Theorem (http://de.wikipedia.org/wiki/Kochen-Specker-Theorem) experimentell bestätigt: Falls dabei das "Wie?" interessiert kann ich leider nicht weiterhelfen ...

richy
23.07.09, 21:39
Hmm versucht Zeilinger nicht gerade das KS Theorem zu wiederlegen ?
Eine Nicht Nichtkontextualitaet ?

@EvB
Das ist die Konseqenz :
B) Es kann, wenn überhaupt, nur nicht-lokale Variablen geben
und nicht-lokal bedeutet global

möbius
24.07.09, 09:06
.....
Ich bin für Spukhafte Fernwirkung:D

Gruß
EVB

Und ich bin für fernhafte Spuck-Wirkung ...:D :D :D
Gruß, möbius

richy
24.07.09, 13:19
Warum nimmt man lieber eine spukhafte Fernwirkung an, als eine globale Variabe ?

Eyk van Bommel
24.07.09, 14:39
Hi richy,

Warum nimmt man lieber eine spukhafte Fernwirkung an, als eine globale Variabe ?
Nun ich verstehe die „spukhafte Fernwirkung“ doch eben als eine „globale Variable“?

Kann man das nicht?

globale Variable bedeutet doch „nur“ (nach meinem bescheidenen Verständnis), dass wenn es eine Informationsausbreitung geben würde (was imho nicht so ist), dann müsste diese mit „v>c“ erfolgen.

Scheinbar zumindest „v>c“ zunächst egal wie – nicht lokal eben?

Gruß
EVB

richy
24.07.09, 15:29
Nun ich verstehe die „spukhafte Fernwirkung“ doch eben als eine „globale Variable“?

Spukhafte Fernwirkung = Dastehen wie der Ochse vorm Berg
Globale Variable = Loesen des Problems durch Hinzufuegen einer weiteren Dimension in der mathematischen Beschreibung

Vielleich erklaert es diesmal jemand anderes. Ich habe mich schon 100 Mal darum bemueht.

Eyk van Bommel
24.07.09, 15:56
Spukhafte Fernwirkung = Dastehen wie der Ochse vorm Berg :D
Globale Variable = Loesen des Problems durch Hinzufuegen einer weiteren Dimension in der mathematischen Beschreibung
Da hast du mich missverstanden.

Ich denke sicher nicht, dass in den Lehrbüchern stehen wird:

Dieses Problem wird mit Hilfe der „Spukhafte Fernwirkung“ gelöst:p

Natürlich wird man eine mathematische Beschreibung dafür finden, die wie du schreibst, durch das einfügen einer weiteren Dimension, gelöst wird(/werden kann?)

Nur verstehe ich das so, das der Lösungsansatz einer weiteren Dimension in der QM (Hilbertraum) ein anderer ist, wie man ihn in der VWT verwendet???

Ich meine in der VWT handelt es sich um „reale“ Dimensionen und in der QM um mathematische?

Aber da lehne ich mich nun auch weit über das hinaus, was ich zu verstehen vermag :o – das ist nur das, was ich mir aus den einzelnen Antworten im laufe der Zeit herausgezogen habe.:o

Daher:
Spukhafte Fernwirkung = Quantenphysikalische Lösung (mit, wenn man so will, einer weiteren QM-Dimension)

Gruß
EVB

PS: In beiden Fällen wird aber imho keine Informationsausbreitung mit v>c erfolgen

Eyk van Bommel
24.07.09, 20:13
Hi richy,

kannst du mir gaaanz kurz den tipp geben, ob ich ganz falsch liege:confused:

Also nicht zum 100- mal erklären - sondern nur

A) Ganz falsch
B) Wenn man so will
C) Kapiere erst gar nicht was du willst

Danke

Gruß
EVB

richy
25.07.09, 04:49
Hi EVB
Hatte gestern nen Gig und heute wieder bald. Dennoch :

Spukhafte Fernwirkung = Ich kann den Vorgang nicht erklaeren.
Es genuegt mir ihn zu messen.
Im Grunde die Einstellung der Kopenhagener Deutung.

Ich meine in der VWT handelt es sich um „reale“ Dimensionen und in der QM um mathematische?
Solch einen Unterschied gibt es allgemein nicht.
Mit der zusaetzlichen Dimension laesst sich die spukhafte Fernwirkung beschreiben ohne irgendwelche anderen physikaischen Gesetze zu verletzen. Das mathematische Modell wird dann global wie die QM.

Man hat einmal das mathematische Modell = abstrakt
dieses beschreibt
den physikalischen Vorgang = physikalisch

Beschreibt das Modell den Vorgang korrekt ist es konsistent.
Welchen physikalischen Bedeutungen die Groessen im Modell zukommen ist die Interpretation.

Bei einer zusaetzlichen globalen Variablen waere die Interpretation zum Beispiel die VWT. Ziemlich zwingend. Aber die wird im Moment gerade genuegend strapaziert.

Es wird auch leider immer nur die Negierung angegeben.
Lokale versteckte Variablen loesen das Problem nicht.
Ja was loest das ERP Problem denn dann ?
Globale Variablen.

richy
25.07.09, 13:13
Hi EvB

Phillip Wehri hat den Vorgang auf seiner Webseite schon beschrieben. Hier nochmal eine kurze nur prinzipielle Fassung wie eine globale Variable die spukhafte Fernwirkung ohne Geister erklaeren kann.

Gegeben sei die zusaetzliche globale Variable x5.
Unsere Realitaet sei willkuerlich dem Wert x5=0 zugeordnet
Wir betrachten noch Welten der willkuerlichen Werte x5=1.1 und x5=1.2

Gegeben sei nun ein Teilchen dass die komplementaeren Zustaende A und B annehmen kann. Die Zustaende sind zunaechst ueberlagert und erscheinen in x5=0 als Projektion, die ich mit U bezeichne. Damit ergibt sich (ASCII) graphisch folgendes Bild :

Ort x ->
X5=0..........U
x5=1.1........A
x5=1.2........B

Nach unten ist x5 aufgetragen , nach rechts der Ort. (BTW A und B sind mitenander verschraenkt)

Nun wird der Zustand U in zwei Zustaende U,U* aufgeteilt die miteinander verschraenkt sind. Es ergibt sich folgendes Bild :

Ort x ->
X5=0..........U..U*
x5=1.1........A..B
x5=1.2........B..A

Die verschraenkten Teilchen entfernen wir etwas voneinander urn den Vorgang etwas spektakulaerer erscheinen zu lassen :

Ort x ->
X5=0..........U.............U*
x5=1.1........A..............B
x5=1.2........B..............A


Nun wird fuer U einer der Zustaende x5=1.1 oder x5=1.2 realisiert.
Wird x5=1.1, U=A realisiert ergibt sich folgendes Bild :
U* muss instantan den Wert U*=B annehmen

Ort x ->
X5=0..........A.............B
x5=1.1........U..............U*
x5=1.2........B..............A


Wird dagegen x5=1.2, U=B realisiert ergibt sich folgendes Bild :
U* muss instantan den Wert U*=A annehmen

Ort x ->
X5=0..........B.............A
x5=1.1........A..............B
x5=1.2........U..............U*

Und schon laesst sich spukhafte Esoterik einfach beschreiben.
Wenn du mal auf einen verschraenkten Bruder von dir stoesst, z.B. bei einer angenommenen Reise in die Vergangenheit, so ist das eine Version von dir aus einer Parallelwelt.
Ich bin uebrigends mit einem richy verschreankt der eine Million EUR im Lotto gewonnen hat und sich gerade auf Hawaii sonnt.
Leider nimmt verschraenkte EUR noch niemand an :-)
D.h. irgendwelche faulen Aktien koennte man als verschraenktes Geld bezeichnen.

richy
25.07.09, 14:17
Anmerkung:
Koennte man in dem einfachen Modell des letzten Beitrags fuer U frei auswaehlen welcher Zustand A oder B realisiert wird, so haette man ein System mit dem Information unendlich schnell uebetragen werden koennte. Falls man dies ausschliesst muss die Auswahl von U physikalisch zufaellig sein. Beides steht also in direktem Zusammenhang.

Uli
25.07.09, 14:40
Hi EvB

Phillip Wehri hat den Vorgang auf seiner Webseite schon beschrieben. Hier nochmal eine kurze nur prinzipielle Fassung wie eine globale Variable die spukhafte Fernwirkung ohne Geister erklaeren kann.

Gegeben sei die zusaetzliche globale Variable x5.
Unsere Realitaet sei willkuerlich dem Wert x5=0 zugeordnet
Wir betrachten noch Welten der willkuerlichen Werte x5=1.1 und x5=1.2

Gegeben sei nun ein Teilchen dass die komplementaeren Zustaende A und B annehmen kann. Die Zustaende sind zunaechst ueberlagert und erscheinen in x5=0 als Projektion, die ich mit U bezeichne. Damit ergibt sich (ASCII) graphisch folgendes Bild :

Ort x ->
X5=0..........U
x5=1.1........A
x5=1.2........B

Nach unten ist x5 aufgetragen , nach rechts der Ort. (BTW A und B sind mitenander verschraenkt)


Ich verstehe mal wieder nur Bahnhof. "A" und "B" sollen "komplementäre" Zustände sein - damit sind wahrscheinlich Eigenzustände gemeint - Zustände, in denen eine Observable scharfe Werte annimmt. Um konkreter zu werden, sagen wir mal, diese Observable sei die z-Komponente des Spins, A wäre dann "Spin Up" und B "Spin Down" und Ü eine Superposition, in der man mit einer gewissen Wahrscheinlichkeit w_up "Spin Up" misst und mit einer Wahrscheinlichkeit (1 - w_up) "Spin Down" - soweit richtig verstanden ?

Nun sagst, du A und B seien miteinander "verschränkt" - wieso das ?
Das sind orthogonale Eigenzustände "Spin Up" und "Spin Down", wieso sollte es da eine Abhängigkeit geben ? Kann dir mal wieder nicht folgen, Richy.

Was ich (und ich behaupte mal, nicht nur ich) unter einer Verschränkung verstehe: eine Verschränkung definiert sich über die Wellenfunktion eines zusammengesetzten Systems - z.B. eines Systems aus 2 Teilchen. Die Nichtlokalität kommt nun ins Spiel, wenn diese 2 Subsysteme örtlich voneinander getrennt sind. Dann bewirkt nach KD die Messung an einem Subsystem, dass nicht nur dieses, sondern auch das andere einen Zustand zu einem scharfen Wert annimmt. Hat man ein statistisches Ensemble solcher Experimente, dann stellt man demzufolge fest, dass die Messungen der Subsysteme korrelieren. Ich sehe in deinen Zuständen A und B nichts von Subsystemen. Wieso sollten A und B - die Eigenzustände zu unterschiedlichen Eigenwerten - etwas mit Verschränkungen zu tun haben ?

Gruß,
Uli

Jogi
25.07.09, 23:40
Hi Uli.




Was ich (und ich behaupte mal, nicht nur ich) unter einer Verschränkung verstehe: eine Verschränkung definiert sich über die Wellenfunktion eines zusammengesetzten Systems - z.B. eines Systems aus 2 Teilchen. Die Nichtlokalität kommt nun ins Spiel, wenn diese 2 Subsysteme örtlich voneinander getrennt sind. Dann bewirkt nach KD die Messung an einem Subsystem, dass nicht nur dieses, sondern auch das andere einen Zustand zu einem scharfen Wert annimmt.
Und genau hier sehe ich das Missverständnis das die KD immer wieder verursacht.

Das andere Subsystem(B) nimmt nicht durch die Messung(an A) einen Zustand ein, sondern die Messung an A teilt uns mit, welchen Zustand B zu diesem Zeitpunkt hat(te).
Die von dir eingebrachte Interpretation nach KD suggeriert dem Unbefangenen die instantane, "spukhafte" Fernwirkung, die es einfach nicht gibt. (Die Betonung liegt auf "Wirkung".)

Die Korrelation zweier miteinander verschränkter Teilchen lässt sich aus der gemeinsamen Emission erklären.
Man darf dabei nur nicht annehmen, dass ein Elektron mit upSpin immer ein Elektron mit upSpin bleibt. Der Spin flippt, das Flippen der Spins ist ab Emission korreliert.


Gruß Jogi

EMI
26.07.09, 02:08
Das andere Subsystem(B) nimmt nicht durch die Messung(an A) einen Zustand ein, sondern die Messung an A teilt uns mit, welchen Zustand B zu diesem Zeitpunkt hat(te).
Die Korrelation zweier miteinander verschränkter Teilchen lässt sich aus der gemeinsamen Emission erklären.
Man darf dabei nur nicht annehmen, dass ein Elektron mit upSpin immer ein Elektron mit upSpin bleibt. Der Spin flippt, das Flippen der Spins ist ab Emission korreliert.
Sehr gut Jogi!

Gruß EMI

richy
26.07.09, 03:39
Hi Uli
Ich weiss nicht was an meinem kleinen Modell nun konkret missverstaendlich sein soll. Denn Du gibst ja ein passendes Beispiel :
A wäre dann "Spin Up" und B "Spin Down" und Ü eine Superposition, in der man mit einer gewissen Wahrscheinlichkeit w_up "Spin Up" misst und mit einer Wahrscheinlichkeit (1 - w_up) "Spin Down" - soweit richtig verstanden ?

Genau. Das waere ein gutes Beispiel. Mit "komplementaer" wollte ich nur ausdruecken, dass es ausser A und B keine weiteren Zustaende gibt.
A oder B sollen auch ganz allgemein irgendeine Eigenschaft eines Teilchens sein.
Man koennte natuerlich noch mehr Zustaende annehmen A,B,C,D... aber ich wollte ja nur das Prinzip moeglichst einfach erlaeutern.

Ort x ->
X5=0..........U
x5=1.1........A
x5=1.2........B

Nun sagst, du A und B seien miteinander "verschränkt" - wieso das ?
Das sind orthogonale Eigenzustände "Spin Up" und "Spin Down", wieso sollte es da eine Abhängigkeit geben ? Kann dir mal wieder nicht folgen, Richy.
Das ASCI Bild stellt wie erwaehnt eine globale Variable x5 dar, die nach unten abgetragen ist. Ist das nicht ersichtlich ? Ich wollte den vorbelasteten Begriff VWT vermeiden. Aber es gibt hier 3 parallele Welten:

X5=0 : Unsere Realitaet
x5=1.1 : Paralellwelt 1 dazu
x5=1.2 : Paralellwelt 2 dazu

In unserer Realitaet wurde im Beispiel der Spin noch gar nicht gemessen. Er liegt daher als ueberlagerter Zustand U vor.
U=unscharf, Ueberlagert
Nehmen wir an A=Spin up. Das Teilchen in diesem Zustand existiert in der Parallelwelt 1.
Und das Teilche mit dem Zustand B=Spin down dementsprechend in der Parallelwelt 2.
Soweit alles klar ?
Diese beiden "Parallelwelt-Teilchen" sind in x5=0 aber noch gar nicht realisiert und erscheinen dort als eine Art Proejektion U. Beschrieben durch eine Wahrscheinlichkeit P dass hier entweder ein Teilchen mit Spin up oder Spin down realisiert wird.
Die Schroedingergleichung beschreibt noch kein realisiertes Teilchen.
Die Wahrscheinlichkeitswelle ist in x5=0 noch kein Teilchen ! Nur in anderen Welten.
Die Teilchen existieren nur in den irrealen aber physikalischen Moeglichkeitswelten hier mit den Koordinaten x5=1.1 und x5=1.2.

Im Grunde sind diese irrealen Teilchen A,B aber keine zwei Teilchen sondern gehoeren zusammen.
Schroedingers tote oder lebendige Katze sind ja auch nicht zwei voellig verschiedene Katzen. Und daher habe ich diese beiden Katzen und auch die Teilchen mit Zustand A und B zweier Parallelwelten als miteinander verschraenkt bezeichnet / definiert.
(BTW A und B sind mitenander verschraenkt)

Vieleicht haette ich diese Bemerkung besser weglassen sollen.
Ich bin mir auch nicht sicher ob diese Charakterisierung im Rahmen einer VWT verwendet wird. Das ist nicht die Verscheaenkung wie sie im EPR Experiment beobachtet wird. Sondern die Verschraenkung der Zustaende eines Teilchens ueber mehrere Parallelwelten. Ich weiss aber nicht ob dieser Begriff im Rahmen einer VWT hier verwendet wird.
Ich wollte damit etwas Spezielles ausdruecken. Das kann man aber auch vergessen. Eine Uebersetzung meines BTW in die KD waere :
Die einzelnen ueberlagerten Zustaende einer Wahrscheinlichkeitswelle sind miteinander "verschraenkt"

Was ich (und ich behaupte mal, nicht nur ich) unter einer Verschränkung verstehe: eine Verschränkung definiert sich über die Wellenfunktion eines zusammengesetzten Systems - z.B. eines Systems aus 2 Teilchen.

Ich bin kein Verschraenkungsspezialist in der Praxis. Kann man die Eigenschaft Spin up, Spin down ueberhaupt verschraenken ?
Aber nehmen wir mal an A und B lassen sich verschraenken.
Deine klassisch verschraenkten Teichen habe ich doch angegeben :

Nun wird der Zustand U in zwei Zustaende U,U* aufgeteilt die miteinander verschraenkt sind. Es ergibt sich folgendes Bild :
Ort x ->
X5=0..........U..U*
x5=1.1........A..B
x5=1.2........B..A

Das waeren also diese zwei von dir klassisch verschraenkten Zustaende U und U*.

Die Nichtlokalität kommt nun ins Spiel, wenn diese 2 Subsysteme örtlich voneinander getrennt sind.

Die verschraenkten Teilchen entfernen wir etwas voneinander urn den Vorgang etwas spektakulaerer erscheinen zu lassen :
ABBILDUNG 3

Ort x ->
X5=0..........U.............U*
x5=1.1........A..............B
x5=1.2........B..............A
Zufrieden ?

Dann bewirkt nach KD die Messung an einem Subsystem, dass nicht nur dieses, sondern auch das andere einen Zustand zu einem scharfen Wert annimmt. Hat man ein statistisches Ensemble solcher Experimente, dann stellt man demzufolge fest, dass die Messungen der Subsysteme korrelieren.
Nach KD ? Aber egal.
Genau deine Beschreibung, die Beobachtung hatte ich in den folgenden zwei ASCI Bildern wiedergegeben.
Aber bleiben wir einfach mal bei bei Abbildung 3
Da gibt es in der Realitaet x5=0 den ueberlagerten Zustand U sowie sein verschraenktes Bruederchen U* in 1 km Entfernung.
Jetzt traktieren wir U mit einer Meßsonde. Wir schliessen U damit an unsere Realiatet an. U muss sich nun entscheiden welchen scharfen Zustand A oder B es annehmen soll. Und damit ob die Parallelwelt x5=1.1 oder x5=1.2 realisiert werden soll. Welche der beiden Welten realisiert wird entscheidet natuerlich der Zufall !
Und der Rest ist im Prinzip selbsterklaerend.
Mit einer globalen Variablen wird die Fernwirkung so trivial, dass es schon wieder fast schwer verstaendlich ist.

Viele Gruesse

richy
26.07.09, 03:49
Das andere Subsystem(B) nimmt nicht durch die Messung(an A) einen Zustand ein, sondern die Messung an A teilt uns mit, welchen Zustand B zu diesem Zeitpunkt hat(te).
Die allgemeine Meinung auch seitens der KD, Z.B. Zeilinger ist, gestuetzt durch Experimente, dass das Teilchen erst durch die Messung seine Eigenschaft erhaelt.
Man muss auch beachten :
Eine EM Welle ist noch kein Teilchen. Das Photon ist das zugehoerige Teilchen.
Man koennte eine EM Welle daher durchaus als einen noch nicht realisierten Zustand betrachten.

EMI
26.07.09, 04:09
Die allgemeine Meinung auch seitens der KD, Z.B. Zeilinger ist, gestuetzt durch Experimente, dass das Teilchen erst durch die Messung seine Eigenschaft erhaelt.
Hallo richy,

durch welche Experimente wird diese "allgemeine Meinung" gestützt?

Gruß EMI

richy
26.07.09, 04:23
Hi Emi
Diese Auesserung habe ich in mehreren "serioesen" Quellen gefunden.
Ich denke selbst bei Zeilinger. Wobei solch ein Verhalten doch eher eine globale Variable, also VWT unterstuetzt.
Aber klar. Kann ich so nicht stehen lassen. Ich versuche die Quellen nachzuliefern.
BTW Vielen Dank fuer deine Rechnung bezueglich der Metrik der ART.
Dazu wollte ich auch noch einiges schreiben.
Momentan laeuft mein Musik Job wieder Mal recht gut.
Das Thema wollte ich aber auf jeden Fall nochmals aufgreifen.

Viele Gruesse
richy

Jogi
26.07.09, 09:49
Sehr gut Jogi!



Daß ich das noch erleben darf...:)


@richy:
Eine EM Welle ist noch kein Teilchen. Das Photon ist das zugehoerige Teilchen.
Man koennte eine EM Welle daher durchaus als einen noch nicht realisierten Zustand betrachten.
Man könnte auch einfach das Photon (oder Elektron), solange es wechselwirkungsfrei unterwegs ist, als in einem (für uns) noch nicht realisierten Zustand bezeichnen.

In unserer Realitaet wurde im Beispiel der Spin noch gar nicht gemessen. Er liegt daher als ueberlagerter Zustand U vor.
U=unscharf, Ueberlagert
Nehmen wir an A=Spin up. Das Teilchen in diesem Zustand existiert in der Parallelwelt 1.
Und das Teilche mit dem Zustand B=Spin down dementsprechend in der Parallelwelt 2.
Soweit alles klar ?
Diese beiden "Parallelwelt-Teilchen" sind in x5=0 aber noch gar nicht realisiert und erscheinen dort als eine Art Proejektion U. Beschrieben durch eine Wahrscheinlichkeit P dass hier entweder ein Teilchen mit Spin up oder Spin down realisiert wird.
Eben.
Es gibt eben nur eine Wahrscheinlichkeit, mit der man das Teilchen A im Zustand Spin up erwischt (durch Messung realisiert).
Bei verschränkten Zuständen (und nur dort) weiß man dann aber instantan, dass gleichzeitig B = Spin down ist.
Die globale Variable ist also nichts anderes als der Messzufall.
Über die Verschränkung ab Emission sollten wir vielleicht mal an anderer Stelle reden.


Gruß Jogi

Eyk van Bommel
26.07.09, 11:11
Hi Jogi,
nach dieser „Theorie“ sollte man doch einen Abstand x, finden andem ich auch für das 2. Photon denselben Spin messe wie beim Ersten?
Spinup =Su
Spindown=Sd

Danach wäre es bei dir so:

-----------------------------------------------------------------------> Räumliche Ausbreitung
Photon 1: Su-Sd-Su-sd- Su-Sd-Su-sd-Su-Sd-Su-sd- Su-Sd-Su-sd- Su-Sd-Su-sd
Photon 2: Sd-Su-sd- Su-Sd-Su-sd-Su-Sd-Su-sd- Su-Sd-Su-sd- Su-Sd-Su-sd-Su
-----------1--2—3—4---5---6---7—8---9—10---11…..
Messe ich A und B immer im gleichen Abstand, dann wäre deine Schlussfolgerung klar.
Messe ich aber z.B A bei 6 und B 9 müsste ich immer ein Photon mit Sd messen!
Und ist das so?

Gruß
EVB
PS: Sorry die Antwort wäre imho zu einfach, so dass diese Theorie einer der Ersten gewesen sein müsste, die man „wiederlegt“ hat?-Oder?

Gandalf
26.07.09, 13:27
Hallo!

Man darf dabei nur nicht annehmen, dass ein Elektron mit upSpin immer ein Elektron mit upSpin bleibt. Der Spin flippt, das Flippen der Spins ist ab Emission korreliert.


Nein, - Eyk hat Recht. Gerade das ist durch die Experimente um die Bellschen Ungleichungen widerelgt

http://de.wikipedia.org/wiki/Bellsche_Ungleichung

Uli
26.07.09, 13:58
Hallo Jogi,

Hi Uli.



Und genau hier sehe ich das Missverständnis das die KD immer wieder verursacht.

Das andere Subsystem(B) nimmt nicht durch die Messung(an A) einen Zustand ein, sondern die Messung an A teilt uns mit, welchen Zustand B zu diesem Zeitpunkt hat(te).


Das ist eben nicht so (s.u.).


Die von dir eingebrachte Interpretation nach KD suggeriert dem Unbefangenen die instantane, "spukhafte" Fernwirkung, die es einfach nicht gibt. (Die Betonung liegt auf "Wirkung".)

Die Korrelation zweier miteinander verschränkter Teilchen lässt sich aus der gemeinsamen Emission erklären.


Das suggeriert nun wieder, dass die Messung am Teilchen A lediglich die Zustände von A und B detektiert - die Drehimpulse, die sie schon bei ihrer Erzeugung bekommen hatten. Die beobachtete Verletzung der Bellschen Ungleichung zeigt aber, dass dies nicht so ist. Der Drehimpuls von A und B nimmt erst durch die Messung einen definierten Zustand an. Das ist nun mal eine Eigentümlichkeit der Quantenmechanik und wenn man der Wellenfunktion unmittelbare physikalische Bedeutung zuschreibt (womit man vorsichtig sein sollte), dann ist dies in der Tat eine spukhafte Fernwirkung.

Die VWT muss in der Lage sein, dieses Feature zu reproduzieren - wozu sie ja auch bekanntlich in der Lage ist, wenn man alles richtig verstanden hat..



Man darf dabei nur nicht annehmen, dass ein Elektron mit upSpin immer ein Elektron mit upSpin bleibt. Der Spin flippt, das Flippen der Spins ist ab Emission korreliert.


Natürlich darf man das annehmen: nach Erzeugung wechselwirken die Teilchen nicht - somit gilt Drehimpuls- und damit Spin-Erhaltung. Der Spin flippt nicht.

Gruß,
Uli

richy
26.07.09, 14:14
Es gibt eben nur eine Wahrscheinlichkeit, mit der man das Teilchen A im Zustand Spin up erwischt (durch Messung realisiert).

Ja. Und diese Wahrscheinlichkeit kann ich bei einem verschraenkten Teilchen nicht durch auessere Einflusse manipulieren. Denn ansonsten koennte ich Information unendlich schnell uebertragen. Entweder geht dies gar nicht oder die Verschraenkung geht dabei verloren. Wie eine Verschraenkung verloren gehen kann, kann ich mit meinem Modell noch nicht erklaeren. Den Fall halte ich daher fuer besonders interessant. Ebenfalls kann ich nicht klaeren wie die Verschraenkung zustande kommt.
Weiterer Fall :
Man kann Information doch unendlich schnell uebertragen. (Schliesse ich eher aus)

Bei verschränkten Zuständen (und nur dort) weiß man dann aber instantan, dass gleichzeitig B = Spin down ist.

Das folgt trivial aus den Skizzen.
Wird fuer U die Welt 1 realisiert wird auch fuer U* die Welt 1 realisiert.x5=1.1 : A..............B
Wird fuer U die Welt 2 realisiert wird auch fuer U* die Welt 2 realisiert.x5=1.2 : B..............A
Denn x5 ist eine globale Variable. Veraendere ich deren Wert kann sich instantan das komplette Universum veraendern. Ich kann aber deren Wert nicht veraendern. Dies ist dem Zufall vorbehalten.

Die globale Variable ist also nichts anderes als der Messzufall.
??? Nein, natuerlich nicht. Die globale Variable ist x5.
Ueber den Zufallsmechanimus der Auswahl kann ich ueberhaupt nichts genaues sagen.
Wenn ich an einem der Teilchen die Messung durchfuehre, dann startet ein Zufallsgenerator, der entweder den Wert x5=1.1 oder x5=1.2 liefert. Und das zugehoerige Universum wird realisiert. Egal welches gewaehlt wird. Die Korrelation war wie man sieht darin schon vorher festgelegt.
Jetzt muesste das Argument kommen, dass es doch zu aufwendig sei nur wegen dem Zustand eines Teilchens ein komplettes Universum zu aendern. Das ist kein Argument, denn man aendert konkret nur einen Zahlenwert.
Und bei den zeitlichen Serielluniveren wundert dies wie immer auch niemanden.
Wenn ich den Zeitpunkt t1 betrachte so liegt dort ein anderes Univesum vor als fuer t2=t1+1 Sekunde.
Nur weil sich der Wert von t etwas geaendert hat.
Was fuer ein Aufwand :-)

PS:
Ist das vereinfachte Modell von mir jetzt eigentlich verstanden ?

Jogi
26.07.09, 17:00
Hi Jogi,
nach dieser „Theorie“ sollte man doch einen Abstand x, finden andem ich auch für das 2. Photon denselben Spin messe wie beim Ersten?
Spinup =Su
Spindown=Sd

Danach wäre es bei dir so:

-----------------------------------------------------------------------> Räumliche Ausbreitung
Photon 1: Su-Sd-Su-sd- Su-Sd-Su-sd-Su-Sd-Su-sd- Su-Sd-Su-sd- Su-Sd-Su-sd
Photon 2: Sd-Su-sd- Su-Sd-Su-sd-Su-Sd-Su-sd- Su-Sd-Su-sd- Su-Sd-Su-sd-Su
-----------1--2—3—4---5---6---7—8---9—10---11…..
Messe ich A und B immer im gleichen Abstand, dann wäre deine Schlussfolgerung klar.
Messe ich aber z.B A bei 6 und B 9 müsste ich immer ein Photon mit Sd messen!
Und ist das so?
Nein, das ist nicht so, wie die Verletzung der Bellschen Ungleichung zeigt.

Wenn wir uns vergegenwärtigen was Spin eigentlich ist (nämlich eben kein klassischer Drehimpuls), dann können wir uns vorstellen, dass der upSpin eine andere Qualität hat wie der downSpin, und dass die Zustände eben nicht genau gleichlang andauern, und deshalb die Korrelation nicht 100% beträgt.

Außerdem kann man ja gar nicht wissen, in welchem Abstand von der Emission der Spin das erste Mal flippt, das hängt von mehreren Faktoren, nennt es meinetwegen verborgene Variablen, ab.


@Uli:
Die Eigenrotation des Teilchens ist nur ein Aspekt des Spins, und sie ändert sich ohne WW natürlich nicht, da hast du freilich Recht.
Dass aber die Richtung des Spins während der WW-freien Phase nicht flippt, kann man gar nicht feststellen. Das Gegenteil zwar auch nicht, aber es macht schon Sinn das anzunehmen, glaub' mir.;)


Wenn ich an einem der Teilchen die Messung durchfuehre, dann startet ein Zufallsgenerator, der entweder den Wert x5=1.1 oder x5=1.2 liefert. Und das zugehoerige Universum wird realisiert. Egal welches gewaehlt wird. Die Korrelation war wie man sieht darin schon vorher festgelegt.
Jetzt muesste das Argument kommen, dass es doch zu aufwendig sei nur wegen dem Zustand eines Teilchens ein komplettes Universum zu aendern.
Genau dieses Argument kommt hiermit.
Das ist kein Argument, denn man aendert konkret nur einen Zahlenwert.
Na also, wo ist das Problem?
Das Teilchen ändert seinen Zustand (Zahlenwert) innerhalb dieses Universums, aber ausserhalb unseres Erfahrungsraumes, weil ohne äussere WW.
Wenn man sagt: Alles was nicht JETZT direkt mit unserem Erfahrungsraum in WW steht, gehört zu einem anderen Universum, dann gibt's aber Paralleluniversen zu Hauf, mein lieber Herr Gesangsverein...

bei den zeitlichen Serielluniversen wundert dies wie immer auch niemanden.
Es wundert mich zwar nicht, aber deshalb muss ich mit dieser Interpretation nicht konform gehen.


x5 ist eine globale Variable. Veraendere ich deren Wert kann sich instantan das komplette Universum veraendern. Ich kann aber deren Wert nicht veraendern. Dies ist dem Zufall vorbehalten.

Die globale Variable ist also nichts anderes als der Messzufall.
??? Nein, natuerlich nicht. Die globale Variable ist x5.
Ueber den Zufallsmechanimus der Auswahl kann ich ueberhaupt nichts genaues sagen.
Deshalb verstehe ich deinen Einwand nicht.

Du sagst selbst, der Wert der globalen Variablen ist zufällig.
Und sinngemäß sage ich das gleiche, der Zustand des Teilchens steht vor der Detektion nicht fest, der Zufall entscheidet über den gemessenen Wert.


Ist das vereinfachte Modell von mir jetzt eigentlich verstanden ?
Ich versteh' ja vieles, aber manchmal fehlt mir das Verständnis.:rolleyes:




Richy, wir könnten (wieder mal) deine Hilfe zu einem ganz anderen Thema gebrauchen, ich melde mich, sobald ich glaube, das Problem so formulieren zu können, das du was damit anfangen kannst.


Gruß Jogi

Uli
26.07.09, 18:35
Hi Richy,



In unserer Realitaet wurde im Beispiel der Spin noch gar nicht gemessen. Er liegt daher als ueberlagerter Zustand U vor.


Wenn ich ehrlich sein darf, Richy: ich habe bisweilen den Verdacht, dass das alles ein wenig durcheinander geht bei dir. So redest du nun von einem überlagerten Zustand in einer der Realitäten. Superpositionen (diese meinst du sicherlich mit "überlagerten Zustand) gibt es doch aber nun gar nicht in der VWT; jeder Basiszustand zur Superposition hat ja in seinem eigenen Unversum einen scharfen Wert.



Im Grunde sind diese irrealen Teilchen A,B aber keine zwei Teilchen sondern gehoeren zusammen.
Schroedingers tote oder lebendige Katze sind ja auch nicht zwei voellig verschiedene Katzen. Und daher habe ich diese beiden Katzen und auch die Teilchen mit Zustand A und B zweier Parallelwelten als miteinander verschraenkt bezeichnet / definiert.


Das hat nichts mit Verschränkung zu tun - Verschränkung ist ein messbarer Effekt.


Vieleicht haette ich diese Bemerkung besser weglassen sollen.
Ich bin mir auch nicht sicher ob diese Charakterisierung im Rahmen einer VWT verwendet wird. Das ist nicht die Verscheaenkung wie sie im EPR Experiment beobachtet wird. Sondern die Verschraenkung der Zustaende eines Teilchens ueber mehrere Parallelwelten. Ich weiss aber nicht ob dieser Begriff im Rahmen einer VWT hier verwendet wird.


Das ist ein unnachweisbares Feature der VWT, das du da beschreibst. Verschränkung aber ist eine experimentell überprüfte Vorhersage der Quantenmechanik. Du solltest es anders bezeichnen; ansonsten wird es immer schwieriger zu folgen. :)

Ich habe aber jetzt verstanden, was du sagen willst, glaube ich.

Gruß,
Uli

Uli
26.07.09, 18:43
@Uli:
Die Eigenrotation des Teilchens ist nur ein Aspekt des Spins, und sie ändert sich ohne WW natürlich nicht, da hast du freilich Recht.
Dass aber die Richtung des Spins während der WW-freien Phase nicht flippt, kann man gar nicht feststellen. Das Gegenteil zwar auch nicht, aber es macht schon Sinn das anzunehmen, glaub' mir.;)
...
Gruß Jogi

Jogi, du scheinst dir so ein Bild zurechtgebastelt zu haben, dass der Spin eines superponierten Zustandes unentwegt hin und her-flippt bis er denn endlich gemessen wird ?

Das ist nicht richtig - damit unterstellst du eine Dynamik, die wir noch nicht kennen; das wäre in der Tat so etwas wie eine Art "hidden variable" Beschreibung des Ganzen. Nach der Quantenmechanik hat der Spin (genauer seine z-Komponente) eines superponierten Zustands keinen wohldefinierten Wert, sondern muss durch eine Wahrscheinlichkeitsverteilung beschrieben werden. Es macht überhaupt keine Sinn von Spin-Flips zu reden.

Gruß,
Uli

Jogi
26.07.09, 18:58
Hi Uli.

Jogi, du scheinst dir so ein Bild zurechtgebastelt zu haben, dass der Spin eines superponierten Zustandes unentwegt hin und her-flippt bis er denn endlich gemessen wird ?
So ist es.
Und nach der Messung geht das geflippe weiter, allerdings ist dann die Korrelation zerstört.


damit unterstellst du eine Dynamik, die wir noch nicht kennen; das wäre in der Tat so etwas wie eine Art "hidden variable" Beschreibung des Ganzen.
Na siehste, geht doch.:)

Nach der Quantenmechanik hat der Spin (genauer seine z-Komponente) eines superponierten Zustands keinen wohldefinierten Wert, sondern muss durch eine Wahrscheinlichkeitsverteilung beschrieben werden.
Sag' ich doch.
Wir wissen nicht, wann der Spin in welche Richtung zeigt.
Erst bei der Messung wird der Wert für uns scharf.
Weil das aber mit einer WW verbunden ist, verschiebt sich die Phase am gemessenen Teilchen, die Korrelation wird aufgehoben.


Es macht überhaupt keine Sinn von Spin-Flips zu reden.
Dan nenn' es halt anders.
Aber das ändert nichts daran, daß dieses Bild in Übereinstimmung mit den Beobachtungen funktioniert.

Ich hätte, wie gesagt, dieses Thema gerne noch vertieft, aber nicht hier.
Erfahrungsgemäß klinkst du dich dann aber stets aus, und das ist schade.


Gruß Jogi

Eyk van Bommel
26.07.09, 21:28
Nein, das ist nicht so, wie die Verletzung der Bellschen Ungleichung zeigt.
Ja, Ja aber das wäre bei dir so ;)
dann können wir uns vorstellen, dass der upSpin eine andere Qualität hat wie der downSpin,
Das würde ich gerne etwas genauer verstanden wissen?
und dass die Zustände eben nicht genau gleichlang andauern, und deshalb die Korrelation nicht 100% beträgt.
Nun für mich liegt die nicht 100% Korrelation eigentlich darin, dass das eine oder andere Photon auf dem Weg zum Detektor Wechselwirkt = gemessen?

Nichts desto trotz würde man in deinem Fall,beim verschieben eines Filters, eine erhöhte Korrelation an einer stelle mit gleichem Spin erwarten?

Man sollte es messen können?

Außerdem kann man ja gar nicht wissen, in welchem Abstand von der Emission der Spin das erste Mal flippt,
Aber es sollte ziemlich kurz sein? Denn sonst hättest du eine, deutlich vom Abstand (Emission und Messpunkt) abhängige Korrelation?

Nach 1 mm 95% und nach 1 km 10%??
das hängt von mehreren Faktoren, nennt es meinetwegen verborgene Variablen, ab.
Das ist der schwierigste Punkt.

Die aber auf 1km in beide Richtungen exakt identisch sind? Also die verborgenen Variablen, lassen „zufällig“ auf km weiter Entfernung die Photonen gleichzeitig flippen/floppen?

Da wären wir ganz nahe bei meiner Vorstellung: Das der "Raum/die Raumzeit" eigentlich verschränkt ist und wir dies mit dem Photon nur messen ;) Aber egal:)
Weil das aber mit einer WW verbunden ist, verschiebt sich die Phase am gemessenen Teilchen, die Korrelation wird aufgehoben.
Also die verborgene Variable wechselwirkt mit den Photonen, dies ist aber eine andere Art der Wechselwirkung wie mit dem Filter, da diese die Korrelation nicht zerstört?
Ich hätte, wie gesagt, dieses Thema gerne noch vertieft, aber nicht hier.
Wo dann?
Gruß
EVB

Jogi
26.07.09, 22:54
Hi EvB.




Nun für mich liegt die nicht 100% Korrelation eigentlich darin, dass das eine oder andere Photon auf dem Weg zum Detektor Wechselwirkt = gemessen?
Wenn ein Photon unterwegs im Sinne einer Messung wechselwirkt, ist da sein Weg zu Ende, es kommt nie am Detektor an.
Womit es ständig wechselwirkt, allerdings ohne Impulskopplung, sind unsere hypothetischen Gravitonen. Das dürfte jedoch auf die Korrelation keinen Einfluss haben, die WW-Rate ist für beide Photonen gleich.


Nichts desto trotz würde man in deinem Fall,beim verschieben eines Filters, eine erhöhte Korrelation an einer stelle mit gleichem Spin erwarten?
Das wäre so, wenn der Spin bei der Emission immer gleich wäre, was aber offenbar nicht der Fall ist.


Denn sonst hättest du eine, deutlich vom Abstand (Emission und Messpunkt) abhängige Korrelation?
Nicht unbedingt, im Gegenteil:
Die up- und downSpins wechseln sich ja zwischen den Teilchen ab, da gleicht sich der Unterschied eher sogar aus.



Außerdem kann man ja gar nicht wissen, in welchem Abstand von der Emission der Spin das erste Mal flippt, das hängt von mehreren Faktoren, nennt es meinetwegen verborgene Variablen, ab.

Das ist der schwierigste Punkt.
Das ist der entscheidende Punkt.

Was auf den folgenden Kilometern passiert, ist eigentlich egal.


Die aber auf 1km in beide Richtungen exakt identisch sind? Also die verborgenen Variablen, lassen „zufällig“ auf km weiter Entfernung die Photonen gleichzeitig flippen/floppen?
Ja. Das hab' ich dir doch schon mal beschrieben.
Die beiden Teilchen haben identische Eigenfrequenzen, da ändert auch die Entfernung erst mal nichts dran. Die Spins wechseln mit dieser Frequenz und sie sind gegenläufig.


Also die verborgene Variable wechselwirkt mit den Photonen, dies ist aber eine andere Art der Wechselwirkung wie mit dem Filter, da diese die Korrelation nicht zerstört?
So ähnlich.
Die verborgene Variable wäre z. B. Richtung und Ort der Spin-Welle (auf dem Teilchen) im Augenblick der Emission.
Das scheint wirklich der pure Zufall zu sein, aber diese Konstellation entscheidet über den Grad der Verschränkung, was im statistischen Mittel eben nicht zu 100% und auch nicht zu 50% Verschränkung führt, sondern zu dem Verhältnis, das diese Gleichung als Mittelwert angibt:
http://upload.wikimedia.org/math/d/d/c/ddc439b02a49ddc65de6c3975c225778.png


dann können wir uns vorstellen, dass der upSpin eine andere Qualität hat wie der downSpin,
Das würde ich gerne etwas genauer verstanden wissen?
Ich hätte, wie gesagt, dieses Thema gerne noch vertieft, aber nicht hier.
Wo dann?
Ich mach mal 'n extra Thread zum Spin auf, ich weiß aber noch nicht, ob ich's heute noch schaffe.


Gruß Jogi

richy
26.07.09, 23:26
Hi Uli
Wenn ich ehrlich sein darf, Richy: ich habe bisweilen den Verdacht, dass das alles ein wenig durcheinander geht bei dir. Superpositionen (diese meinst du sicherlich mit "überlagerten Zustand) gibt es doch aber nun gar nicht in der VWT; jeder Basiszustand zur Superposition hat ja in seinem eigenen Unversum einen scharfen Wert.
Kann sein, dass ich den Begriff komplementaer auf Philllp Wehrlis Seite falsch verstanden habe.
Nimmt das verschraenkte Teilchen die selbe Eigenschaft an wie sein Zwilling oder eine Komplementaere ?

2. Das Experiment

Es gibt verschiedene Varianten des EPR-Experimentes. Die folgenden Elemente sind in allen Varianten gleich:
- Zwei Teilchen werden miteinander verschränkt oder so erzeugt, dass sie miteinander verschränkt sind (was das genau bedeutet, wird unten erklärt).
- Die Teilchen werden an zwei verschiedene Orte gebracht.
- An den Teilchen werden komplementäre Grössen gemessen.
- Dieser Vorgang wird mit mehreren Teilchenpaaren wiederholt. Die statistische Auswertung aller Messresultate zeigt ein Ergebnis, das mit keiner lokal-realistischen Theorie erklärt werden kann.

http://homepage.hispeed.ch/philipp.wehrli/Physik/Quantentheorie/Einstein_Podolsky_Rosen/einstein_podolsky_rosen.html
Sind die korrelierten Groessen gleich waere dies aber auch kein Beinbruch.
Statt
Ort x ->
X5=0..........U..U*
x5=1.1........A..B
x5=1.2........B..A

ergaebe die Verschraenkung
Ort x ->
X5=0..........U..U*
x5=1.1........A..A
x5=1.2........B..B

Der restliche Ablauf wie gehabt.
Ich meine fast ich habe den Begriff komplementaer hier falsch verstanden. Sorry.
Er meinte wohl komplementaer im Sinne der Unschaerfe. Ort/Impuls.
Sollte ich korrigieren.

Mein Modell kann in der einfachen Form auch nur eine 100% ige Korrelation erklaeren.


WIKI
Unter Superposition (lat. super = über; positio = Lage, Setzung, Stellung) versteht man generell eine Überlagerung.

Superposition klingt vielleicht wichtiger, aber Ueberlagerung ist dafuer der deutsche Ausdruck und damit verstaendlicher. Ich benutze auch meist nicht den Begrff Computer sondern Digitalrechner oder Rechner.
Ganz bewusst.
So redest du nun von einem überlagerten Zustand in einer der Realitäten.

Es gibt keine verschiedenen Realitaeten, sondern fuer uns nur eine.

Superpositionen (diese meinst du sicherlich mit "überlagerten Zustand) gibt es doch aber nun gar nicht in der VWT
Wie kommst du darauf, dass es die in einer VWT nicht gibt ? Blos weil die Katze nicht verschmiert ist ?
Deren Kasteninneres gehoert nicht zu unserer Realitaet. Per Defenition. Der Kasten ist voellig von unserer Welt isoliert.
Sie lebt dort in zwei zu uns irrealen Welten. In einer ist sie tod und in der anderen quicklebendig. Das waere die Vorstellung die man direkt aus der mikroskopischen Welt uebernimmt.
Man kann fuer makroskopische Faelle auch annehmen, dass die Katze in ihrem kleinen Kastenuniversum staendig in ihrer Katzenrealitaet bleibt und dort eine Verzweigung stattfindet. Wobei die Katzenrealitaet seinen Verlauf nimmt sobald das verhaengnisvolle Teilchen realisiert ist oder eben nicht.
Das EPR Experiment findet aber nicht in einem Kasten statt und es benutzt keine verschraenkten Katzen sondern mikroskopische Teilchen. Und daher muss der Vorgang so beschrieben werden wie ich ihn beschrieben habe.
Ueberleg doch mal. Ein stets scharfer Zustand in unserer Realitaet waere ein voelliger Widerspruch zur Schroedingergleichung.
Gilt in der VWT die Schroedingergleichung etwa nicht ?
Das waere uebel.

Du machst den Fehler, das du denkst, dass wegen der Katze es bei der VWT immer einen scharfen Zustand in der Realitaet gibt.
Und diese Verzweigungsdiagramme tragen dazu ihren Teil bei.
Ich habe auch keine David Deutsch VWT beschrieben sondern eine Welt mit einer zusaetzlichen globalen zeitartigen Variablen. So wie im Heim Modell.
jeder Basiszustand zur Superposition hat ja in seinem eigenen Unversum einen scharfen Wert.

Na was hab ich hier denn skizziert.
Zwei Zustaende die in ihren Welten einen scharfen Wert haben.

Ort x ->
X5=0..........U
x5=1.1........A
x5=1.2........B

Statt U kannst du auch S wie Schroedingergleichung schreiben.
Oder gar nix hinschreiben. Das waere aber zu wenig.

Das ist ein unnachweisbares Feature der VWT, das du da beschreibst. Verschränkung aber ist eine experimentell überprüfte Vorhersage der Quantenmechanik. Du solltest es anders bezeichnen; ansonsten wird es immer schwieriger zu folgen.

Ok, ich will auch nochmal schauen wie Prof Zeh den Begriff verwendet.

Gruesse

richy
27.07.09, 00:09
@Jogi EMI
Wenn man eine groessere Anzahl verschiedener verschraenkter Teilchen die sich voneinander fortbewegen messen wuerde. In verschiedenen Abstaenden der Teilchen zueinander misst. So ergaebe sich dein scheinbares Flip Flop Bild.
Das erklaert aber nichts. Ueber welchen Mechanismus sind die Teilchen korreliert ? Bei mir ist der Mechanismus die globale Variable. Wobei mein Modell zu einfach ist um alle Mechanismen zu erklaeren. Aber immerhin ist damit die Fernwirkung vom Tisch.
BTW: Bei dem Begriff komplementaer ist mir scheinbar ein Fehler unterlaufen. Siehe oben.
Und mein Modell erklaert vereinfacht nur eine 100% ige Korrelation.

Zeilingers Meinung ;
Das Faszinierende ist, dass uns die Quantenphysik sagt, dass es Dinge gibt - der berühmte quantenmechanische Einzelprozess -, die sich dann letztlich rein zufällig verhalten. Das heißt, dass die Möglichkeit, da irgendwelche Gedankenketten zu konstruieren, und seien sie auch noch so kompliziert, nicht funktioniert. Das heißt, es gibt Dinge, die geschehen ohne Grund. Das ist für mich die wichtigste Erkenntnis des 20. Jahrhunderts. Das Interessante ist, dass man dieses Faktum im Experiment wirklich sehen kann, dass man es nicht bloß indirekt über theoretische Schlüsse konstruieren muss.

Um die Quantenteleportation erklären zu können, muss ich den Begriff der Verschränkung kurz erwähnen. Den muss man einmal verstanden haben, das ist eine wichtige Voraussetzung. Also, ich kann eine Situation haben, wo ich mir zwei Teilchen erzeuge, an denen ich dann Messungen, Beobachtungen vornehme. Jede Messung kommt einer Frage an die Natur gleich: "Welche Eigenschaften hast du?"

Im Zuge dieser Beobachtungen - sagen wir, wir messen die Polarisation, die Schwingungsrichtung - stelle ich fest, dass beide Teilchen identisch sind. Die ursprüngliche Annahme war die, dass sie deshalb identisch sind, weil sie identisch erzeugt, mit den gleichen Eigenschaften geboren wurden, wie etwa eineiige Zwillinge. Das Verrückte ist jetzt, dass wir heute auf Grund von Experimenten wissen, dass diese Erklärung falsch ist.

Die einzige andere mögliche Erklärung ist, dass die Messung an einem dieser beiden Teilchen das andere Teilchen in irgendeiner Form beeinflusst, ganz egal wie weit es entfernt ist. Das ist diese Art von "spukhafter Fernwirkung", von der Einstein nichts wissen wollte. Dieses eigenartige Verhalten nennt man Verschränkung. Die beiden Teilchen haben vor der Beobachtung keine Eigenschaft. In dem Moment aber, wo ich eines von ihnen beobachte, nimmt es eine Eigenschaft an, und das andere Teilchen nimmt die gleiche Eigenschaft an, ganz egal, wie weit die beiden Teilchen voneinander entfernt sind.
Genau wie es mein einfaches Modell erklaert.
Warum haben sie vor der Messung keine Eigenschaft ? Weil sie noch nicht realisiert sind.
Sie befinden sich noch in Parallelwelten aber schon mit den entsprechenden Korrelationen.

http://www.philosophische-praxis.at/zeilinger.html

richy
27.07.09, 01:25
Hier nochmal die Version wenn die Teilchen identische Zustaende annehmen. Wie gesagt ich bin kein praktischer Verschraenkungsspezialist. Das ist ein rein prinzipielles Modell :


Ort x ->
X5=0..........S
x5=1.1........A
x5=1.2........B

Nach unten ist x5 aufgetragen , nach rechts der Ort.

Nun wird der Zustand S in zwei Zustaende S,S* aufgeteilt die miteinander verschraenkt sind. Es ergibt sich folgendes Bild :

Ort x ->
X5=0..........S..S*
x5=1.1........A..A
x5=1.2........B..B

Die verschraenkten Teilchen entfernen wir etwas voneinander urn den Vorgang etwas spektakulaerer erscheinen zu lassen :

Ort x ->
X5=0..........S.............S*
x5=1.1........A..............A
x5=1.2........B..............B


Nun wird fuer S einer der Zustaende x5=1.1 oder x5=1.2 realisiert.
Wird x5=1.1, S=A realisiert ergibt sich folgendes Bild :
S* muss instantan den Wert S*=A annehmen

Ort x ->
X5=0..........A.............A
x5=1.1........X..............X*
x5=1.2........B..............B


Wird dagegen x5=1.2, S=B realisiert ergibt sich folgendes Bild :
S* muss instantan den Wert S*=B annehmen

Ort x ->
X5=0..........B.............B
x5=1.1........A..............A
x5=1.2........X..............X*

richy
27.07.09, 09:22
Hier noch eine Version mit einer ueber die Wahrscheinlichkeit P(x5) einstellbaren Korrelation

Ort x ->
X5=0..........S..S*
x5=1.1........A..A
x5=1.2........B..B
x5=1.3........A..B
x5=1.4........B..A

Uli
27.07.09, 09:52
Hier noch eine Version mit einer ueber die Wahrscheinlichkeit P(x5) einstellbaren Korrelation

Ort x ->
X5=0..........S..S*
x5=1.1........A..A
x5=1.2........B..B
x5=1.3........A..B
x5=1.4........B..A

Hmm, wo bleibt denn da die Verschränkung ?

Verschränkung bedeutet doch: eine Messung am Ort 1 legt das Ergebnis am Ort 2 fest. Das schaut bei dir gar nicht danach aus. Du hast Welten, in denen du am Ort 1 A (Spin Up) und am Ort 2 A (Spin Up) hast als auch solche, in denen du am Ort 1 A (Spin Up) und am Ort 2 B (Spin Down) hast. Da sehe ich keine Verschränkung.

Nehmen wir mal an, in unserem Beispiel sei die Verschränkung über Gesamtdrehimpuls=0; dann wären nur

Ort x ->
x5=1.3........A..B
x5=1.4........B..A

zulässige Kombinationen, da die z-Komponenten der Spins sich jeweils zu 0 aufaddieren. AA und BB wären durch Dreimpulserhaltung ausgeschlossen, d.h. es gibt keine Welt, die so eine Kombination realisiert - sie würde ja einen Erhaltungssatz verletzen.

Sorry, dass ich da so penetrant nachfrage, aber die Beschreibung der Verschränkung ist doch gerade das Thema hier.

Gruß,
Uli

möbius
27.07.09, 10:00
......
1. Globale Variable = Loesen des Problems durch Hinzufuegen einer weiteren Dimension in der mathematischen Beschreibung

2. Vielleich erklaert es diesmal jemand anderes.
3. Ich habe mich schon 100 Mal darum bemueht.
Hallo richy!
Zu 1.:
Gibt es zu dieser sog. "globalen Variablen" auch einen empirisch-experimentellen Zugang :confused:
Zu 2.:
Hoffentlich bald...:D
Zu 3.:
"Wer immer strebend sich bemüht...";)
Gruß, möbius

richy
27.07.09, 10:48
Hi Uli

Kann man den Spin ueberhaupt verschraenken ?
Falls du meine letzten Threads gelesen hast. Ich bin mir nicht mehr sicher wie sich zwei verschraenkte Teilchen nun in speziellen Experimenten konkret verhalten. Im Beispiel mit den Bellschen Ungleichungen wird die Polarisation verwendet. Da scheinen die Zwillinge sich gleich und nicht gegensaetzlich zu verhalten. So beschreibt dies auch Zeilinger. Ebenso ibetraegt die Korrelation zwischen den Zwillingen nicht 100 %.
Es war anfangs gar nicht mein Anspruch dies genau zu modellieren. Mein Modell sollte lediglich die Nichtkokalitaet erklaeren. In der obigen erweiterten Form laesst sich nun auch der Grad der Korrelation einstellen.
Auch das Polarisationsbeispiel fuer die Bellschen Ungleichungen.
A= Photon passiert
B= Photon passiert nicht

Fuer mich gibt es kaum einen Zweifel, dass ein objektiver Zufall existiert. Diesen werden wir in naher Zukunft nicht erklaeren koennen. Und in diesen habe ich nun erstmal alle Problematik verschoben.

Ein wichtiger Teil in meinem letzten Beitrag ist :
mit einer ueber die Wahrscheinlichkeit P(x5) einstellbaren Korrelation
Wie diese Einstellung der Wahrscheinlichkeiten konkret zustande kommt lasse ich zunaechst mal offen.
Im naechsten Beispiel gebe ich noch die Wahrscheinlichkeiten fuer die Realisation jeder Welt an. Damit sollte das Prinzip klar werden :
(Gemessen wird das linke Teilchen)
A= Photon passiert
B= Photon passiert nicht

Was haettens denn gerne ?

Das Photon soll immer den Filter passieren und ebenso der Zwilling :

Ort x ->
X5=0...............
x5=1.1........A..A, P=1
x5=1.2........B..B, P=0
x5=1.3........A..B, P=0
x5=1.4........B..A., P=0

Zustand nach Messung
x5=0...........A..A


Die Photon sollen zu 100% verschraenkt, korreliert sein. Zu 50% den Filter passieren

Ort x ->
X5=0...............
x5=1.1........A..A, P=0.5
x5=1.2........B..B, P=0.5
x5=1.3........A..B, P=0
x5=1.4........B..A., P=0

Zustand nach Messung
x5=0...........A..A oder
x5=0...........B..B

Die Korrelation soll 80% betragen

Ort x ->
X5=0...............
x5=1.1........A..A, P=0.4
x5=1.2........B..B, P=0.4
x5=1.3........A..B, P=0.1
x5=1.4........B..A., P=0.1

Bei 100 Photonen liegt etwa 80 mal der Fall vor :
x5=0...........A..A oder
x5=0...........B..B
und 20 mal
x5=0...........A..B oder
x5=0...........B..A

Das verschrankte Teilchen soll immer die komplementaere Eigenschaft annehmen
Ort x ->
X5=0...............
x5=1.1........A..A, P=0
x5=1.2........B..B, P=0
x5=1.3........A..B, P=0.5
x5=1.4........B..A., P=0.5

Jeder Fall ist nun "frei programmierbar"


Gibt es zu dieser sog. "globalen Variablen" auch einen empirisch-experimentellen Zugang

Ja, die Quantenmechanik Schroedingergleichung.


Viele Gruesse

Uli
27.07.09, 11:21
Hi Uli

Kann man den Spin ueberhaupt verschraenken ?


Selbstverständlich kann man das.


Falls du meine letzten Threads gelesen hast. Ich bin mir nicht mehr sicher wie sich zwei verschraenkte Teilchen nun in speziellen Experimenten konkret verhalten. Im Beispiel mit den Bellschen Ungleichungen wird die Polarisation verwendet.

Viele Gruesse

Die Zustände "Spin Up" und "Spin Down" eines Elektrons sind komplementär (würde Phillip sagen): sie ergeben sich durch die Projektion des Teilchen-Spins auf die z-Richtung. Das ist übrigens etwas ganz ähnliches wie die Projektion des Photon-Spins auf seine Bewegungsrichtung ("Polarisation").
Drehimpulserhaltung würde solche Zustände 2er Subsysteme verschränken.

Gruß,
Uli

richy
27.07.09, 23:45
Hi Uli
Kann man den Spin ueberhaupt verschraenken ?
Selbstverständlich kann man das.


Ich habe das nicht als selbstverstaendlich angenommen. Gerade wegen dem von dir genannten Gesamtdrehimpuls. Dazu haette ich noch ein paar Fragen. Angefangen bei der Bellschen Ungleichung.
Artikel ueber die Bellsche Ungleichunge habe ich schon etliche gelesen, aber so ganz vollstaendig verstehe ich deren Aussage noch nicht.

Es gibt ja zweierlei Dinge ueber die man sich bei der Quantenmechanik wundern soll.
1) Die Nicht Lokalitaet (Globalitaet)
2) Die Nicht Realitaet

Ueber Punkt 1 wudere ich mich schon lange nicht mehr.
Fuer Punkt 2 wird im Link zu der Bellschen Ungleichung als Definition folgendes angegeben :
http://de.wikipedia.org/wiki/Bellsche_Ungleichung
Eine physikalische Theorie ist real, wenn jede Messung nur eine Eigenschaft abliest, die auch ohne Messung vorliegt, wenn also der Wert jeder denkbaren Messung feststeht, selbst wenn wir ihn wegen ungenügender Kenntnis verborgener Parameter nicht vorher wissen.

Dass die Quantenmechanik nicht real ist wundert mich natuerlich nicht, da ich mir dies ebenfalls uber eine globale Variable erklaeren kann.
Aber was ich nicht verstehe ist inwiefern die Bellschen Ungleichungen vermitteln sollen, dass die Quantenmechanik diese Definition der Realitaet nicht erfuellen.

Ich sehe hier zunaechst zwei primaere Punkte :

A) Objektiver Zufall
***************
Kann man aufgrund der Bellschen Ungleichung darauf schliessen, dass es einen objektiven Zufall gibt ? Beweisen kann man diesen meiner Meinung nach so gut wie nicht. Ich sehe hier auch keinen Zusammenhang zur Bellschen Ungleichung.

B) Keine 100% Korrelation
*******************
Als Conclusion dass die Quantenmechanik die Bellsche Ungleichung nicht erfuellt wird (fuer mein Verstaendnis) betont, dass das Experiment zeigt, dass die Verschraenkung zweier Teilchen keine 100% Korrelation aufweist.
Ist dies zutreffend. Also dass dies eine Begruendung fuer 2 ist ?
Mit dem Abschnitt "Quantenmechanische Mittelwerte" im Link
http://de.wikipedia.org/wiki/Bellsche_Ungleichung
komme ich einfach nicht zurecht.

Ich verstehe nicht wie hier die Nicht Realitaet belegt sein soll.
Kann man das Ergebnis in einfache Worte fassen, vielleicht anhand eines anderen anschaulichen Beispiels ?
Objektiver Zufall und Nicht Realitaet sind fuer mich so selbstverstaendlich, dass ich vielleicht der Schlussfogerung einfach nicht folgen kann.
Eine Erklaerung waere nett :-)

Zurueck zum verschraenkten Spin und der Bellschen Ungleichung :
Ein Polarisationserhaltungssatz ist mir nicht bekannt.
Aber natuerlich ein klassischer Drehimpulserhaltungssatz.

Wenn ich unter dem Aspekt der (experimentell belegten) Nicht Realitaet zwei verschraenkte Teilchen betrachte ...
Na eigentlich muesste ich noch einfacher anfangen :
Welchen Spin hat eigentlich die Schroedingerwelle eines Elektrons ?
...
Also lieber doch der verschraenkte Fall :
Wenn die realisierten Spins komplementaer sind, im Sinne von gegensaetzlich, sich zum Ganzen zu ergaenzen (Die Komplementaerfarben der Malerei lassen gruessen) dann ist der Gesamtspin des verschraenkten Systems immer Null. Falls die Korrelation 100% betraegt.
A sei spin up.
B sei spin down
A............B Gesamtspin =0
B............A Gesamtspin =0

Wenn die Korrelation nicht 100% betraegt. Was waere dann ?
A............A Gesamtspin = k
B............B Gesamtspin = -k

Temporaer waere der klassische Drehimpulserhaltungssatz verletzt sobald die Korrelation gestoert ist.
Das darf im Grunde nicht sein.
Treten (A,A) und (B,B) gleich haeufig auf waere der klassische Drehimpulserhaltungssatz zumindestens im zeitlichen Mittel erfuellt.

Wie sieht dies im konkreten Experiment aus ?

Viele Gruesse

richy
28.07.09, 00:57
Hi Moebius
Sorry fuer meine praegnante und daher so kurze Antwort.
Ok, ich gehe nochmal deine Fragen durch :
Sorry im Voraus wenn ich dabei vielleicht etwas zu persoenlich werde.

Globale Variable = Loesen des Problems durch Hinzufuegen einer weiteren Dimension in der mathematischen Beschreibung

Gibt es zu dieser sog. "globalen Variablen" auch einen empirisch-experimentellen Zugang

Zunaechstmal impliziert deine Frage fuer mich eine Aussage :
Globale Variablen (VWT) sind experimentell nicht belegt.
Das ist ein geradezu irrwitziges und dennoch staendig angefuehrtes scheinbares Gegenargument fuer eine globale Variable (VWT)
Diese frage ist so idiotisch (sorry) dass sie bei haeufiger Wiederholung geradezu zu einer Ermuedung und Resignation fuehrt.
Deine Frage stellt auch noch die Schroedingergleichung in Frage.

Warum sind die Beobachtungen bei Experimenten der Quantenmechanik denn so seltsam fuer uns ?
Dazu muss man erstmal anhand von Experimenten erkennen was denn hier so seltsam ist.
Im Rahmen der Bellschen Ungleichungen sind die zwei wichtigsten Punkte angefuehrt :
http://de.wikipedia.org/wiki/Bellsche_Ungleichung
2.) Eine physikalische Theorie ist lokal, wenn sich bei zwei räumlich weit getrennten Teilchen die Wahl dessen, was beim einen Teilchen gemessen wird, nicht augenblicklich auf das andere Teilchen auswirkt.

Wie immer die Negierung. Die eigentliche Aussage lautet :
2.) Eine physikalische Theorie ist global, wenn sich bei zwei räumlich weit getrennten Teilchen die Wahl dessen, was beim einen Teilchen gemessen wird, sich augenblicklich,instantan auf das andere Teilchen auswirkt.

Und ganau dies wird in der Quantenmechanik beim EPR Experiment beobachtet.
Keine Lokalitaet sondern eine Globalitaet !

Es gibt in der Mathematik oder Informatik lokale und globale Parameter und lokale und globale Variablen.
Nicht lokal ist ein Kunstwort ! Das bedeutet "GLOBAL" GLOOOOOOBAAAALLL !
Zusaetliche Dimension.
Die Quantenmechanik ist global !
DAS EPR EXPERIMENT ERLAUBT HIER KEINERLEI WIDERSPRUCH.
Es gibt ueberhaupt keinen Zweifel daran dass wir in einem globalen Universum leben. Das ist tausendmal experimentell belegt.
Der Wiki Text vermeidet es peinlichst das Wort "global" zu verwenden und benutzt stets die Negierung "Nicht lokal" ="global".
Warum wohl ?
Weil ein globales Universum wie es DAS EPR EXPERIMENT EXPERIMENTELL BELEGT ein Multiversum ist.
Schliesslich fuert man im Wiki Link noch den unsinnigeren Begriff "verborgener Parameter,Variablen ein." Parameter oder Variablen sind mathematische, abstrakte Objekte die sich nicht verbergen. "Verborgen" ist eine physikalische Eigenschaft.

Das Ganze gipfelt dann in einem recht unverstaendlichen, voellig sinnlosen Satz :
Daher kann die Quantenmechanik nicht durch Hinzufügen von verborgenen Variablen zu einer realen und lokalen Theorie vervollständigt werden.

Ich wandle diese Aussage mal in sinnvolle Saetze um :
Unter dem experimentellen Aspekt der auf der "Bell Seite" auch wiedergegeben ist und da lautet :
Die Quantenmechanik ist keine reale und lokale Theorie.
Weil es so schoen ist gleich nochmal :
Die Quantenmechanik ist keine reale und lokale Theorie.
und nochmal
Die Quantenmechanik ist eine irreale und globale Theorie.
Und sie ist ein physikalisch tausendmal belegter Fact !
Daher kannst du dir dein Belegbarkeitsargument vergessen.

Sinnvolle Aussagen waeren also :
Daher kann die Quantenmechanik nicht durch Hinzufügen von lokalen Variablen zu einer realen und lokalen Theorie vervollständigt werden.

Das ist der Abgesang an die Kleingeister die meinen man koennte die Quantenmechanik irgendwie reparieren, damit sie auch fuer Lieschen Mueller verstaendlich wird. Wobei man Lieschen Mueller nicht unterschaetzen sollte.
EDIT : Gekuerzt

Eine weitere experimentell konditionierte Aussage :
Daher kann die Quantenmechanik nur durch Hinzufügen von globalen Variablen zu einer globalen Theorie vervollständigt werden. Die Nicht Realitaet muss ueber diese globale Variale erklaert werden koennen.


Schon mein Primitiv Modell leistet beides. Erklaert die Nicht Lokalitaet, Fernwirkung, die Bellsche Ungleichung.
Den Mechanismus der Weltenauswahl (Zusammebruch der Ueberlagerung) erklaert mein Modell nicht.
Und natuerlich keinesfalls den objektiven Zufall.
Das Heim Modell ist eine Anleitung wie man alles erklaeren koennte.
Physikalische und abstrake Welt. Sogar den Urgrund vor dem Urknall.
Aber das Programm des Zufalls kann auch Heim nicht erklaeren.
Die Mechanismen schon.
Warum kann Heim das Programm nicht erklaern ?
Weil dieses das Gesamtprogamm des Universums ist !
Wie kommt diese Programm zustande ?
Weil das Universum in unglaublich feinen Abstaenden sich selbst abtastet und diese Information an G4 weiterleitet.
Ungeheuerliche Vorstellung nicht :-)
Was sind denn die Sensoren fuer diese Messung ?
Jedes Elemetarteilchen !
So ein Quatsch plappert der Kopenhagener der wie selbstverstaendlich die abstrakte Welt mit einbezieht ohne nichteinmal in der Lage ist sein bewusstes Dasein zu quantifizieren.

Soweit alles klar ?

Vielleich erklaert es diesmal jemand anderes.
Hoffentlich bald...

Kandidaten waeren :
Gandalf, Phillip Wehli, Hermes,
auch
Uli, Jogi, Timm, dandy ... (sorry wenn ich jemanden vergessen habe)
EMI haelt sich zurueck weil er der Quantenmechnaik erst gar nicht in die Augen schauen will.
Das steht jedem frei. Ich meine aber es lohnt sich diesen Weg naeher zu betrachten.
Beim Mainstream wie Herrn Lesch oder Herrn Bauhaus Herrn pauli wirst du warten bis du schwarz oder global wirst.

"Wer immer strebend sich bemüht..."
Macht nichts verkehrt, wenn die Motivation seines Strebens die reine Neugier ist.


Viele Gruesse

Uli
28.07.09, 09:39
Temporaer waere der klassische Drehimpulserhaltungssatz verletzt sobald die Korrelation gestoert ist.
Das darf im Grunde nicht sein.
Treten (A,A) und (B,B) gleich haeufig auf waere der klassische Drehimpulserhaltungssatz zumindestens im zeitlichen Mittel erfuellt.

Wie sieht dies im konkreten Experiment aus ?

Viele Gruesse

Drehimpulserhaltung gilt auch quantenmechanisch exakt, d.h. AA oder BB kann man nie beobachten.
Für die restlichen Fragen gib mir etwas Zeit - müsste mir erst man den text im Wiki "antun".

Gruß,
Uli

Bauhof
28.07.09, 11:00
Beim Mainstream wie Herrn Lesch oder Herrn Bauhaus Herrn pauli wirst du warten bis du schwarz oder global wirst.

Hallo Richy,

falls du mit "Bauhaus" mich meinst, dann bedanke ich mich bei dir, dass du mich in einem Atemzug mit Prof. Harald Lesch nennst. Aber das wäre der Ehre zuviel, mit dem kann ich mich nicht vergleichen.

M.f.G. Eugen Bauhof

P.S.
Und dein ewiges Gefasel vom "Mainstream" wirkt bei mir schon langsam etwas komisch. Müssen wir tatsächlich jeden sprachlichen und sonstigen Mist, der aus den USA kommt, kritiklos übernehmen?

rene
28.07.09, 18:15
Wird ein Polarisationsfilter parallel zur Polarisationssebene eines Photons gedreht, so kommt es mit einer Wahrscheinlichkeit von Eins auf jeden Fall durch; wird der Polarisationsfilter um 90° gedreht, wird es auf jeden Fall absorbiert. Für beliebige Drehwinkel zwischen Polarisationsfilter und Polarisationsebene beträgt die Wahrscheinlichkeit für ein Durchkommen des Photons durch den Filter

p = cos²(θ)

Ausser für die genannten Spezialfälle mit θ = 0°, 90°, 180°, 270° steht das Ergebnis erst mit der Messung fest. Dabei wird die ursprüngliche Polarisationsebene des emittierten Photons mit der Wahrscheinlichkeit p in die Ebene des Polarisationsfilters gedreht – resp. mit der Gegenwahrscheinlichkeit q = 1 - p um 90° dagegen verdreht und somit blockiert.

Diese Grundannahme der Verdrehung ist wichtig um sich die Verschränkung polarisierter Photonenpaare zu vergegenwärtigen. D.h. von einem verschränkten Photonenpaar soll je ein Photon nach links und und das andere nach rechts emittiert und im gleichen Abstand vom Zentrum von je einem Polarisationsfilter gemessen werden, die zunächst einen beliebigen, jedoch identischen Polarisationswinkel von

θ1 = θ2

haben sollen. In jedem Fall werden beide Photonen entweder durchkommen oder hängenbleiben. Dies gilt für sämtliche Photonenpaare, was bereits die Lokalität und Realität der Quantenmechanik in Abrede stellt, da vor der Messung die Polarisationsebene des Photons nicht feststeht und nach der Messung (der Polarisationsfilter dient als “Messgerät“) die ursprüngliche Polarisationsrichtung des Photons nicht reproduziert werden kann.

Die Polarisationsebenen der beiden verschränkten Photonen befinden sich in einem identischen quantenmechanischen Zustand. Ist die eine Polarisationsebene unbestimmt, so ist es auch diejenige des verschränkten Partnerphotons. Werden beide gemessen, so liefern sie identische Ergebnisse. Entweder beide kommen durch oder beide bleiben hängen. Die Nicht-Lokalität der verschränkten Photonen – sie haben ja keine Möglichkeit des Signalaustausches – zeigt sich noch mehr wenn verschiedene Einstellungen der beiden Polarisationsfilter vorgenommen werden mit

θ1 ≠ θ2

Die verschränkten Photonen werden sich mit einer gewisssen Wahrscheinlichkeit nicht mehr gleich verhalten. Wenn wir annehmen, das eine des Photonenpaars komme durch den 1. Polarisationsfilter (mit p= cos²(θ1)), so ist seine gemessene, jedoch von der ursprünglichen und unbekannten Ausrichtung abweichende Polarisationsebene mit θ1 bestimmt und somit auch die Polarisationsebene von θ1 des anderen Photons, das einen Polarisationswinkel von θ1 - θ2 aufspannt und mit der entsprechenden Wahrscheinlichkeit von

p(θ2) = cos²(θ1 - θ2)

durchkommt. Die Wahrscheinlichkeit, dass beide Photonen durchkommen, ist das Produkt von p(θ1) = cos²(θ1) und p(θ2) = cos²(θ1 - θ2), also

p(θ1, θ2) = cos²(θ1) * cos²(θ1 - θ2)

und ist symmetrisch in Bezug auf Vertauschen der beiden Winkel θ1 und θ2.


Für den Fall dass eines durchkommt und das andere nicht, gilt analog:

p(θ1, non θ2) = cos²(θ1) * sin²(θ1 - θ2)

und ist wiederum symmetrisch in Bezug auf Vertauschen der Winkel Winkel θ1 und θ2.

Als Fazit lässt sich aus meiner Sicht nur noch folgendes sagen: Gäbe es eine lokal-realistische Theorie, stünde sie im krassen Widerspruch zur Quantenmechanik. Siehe auch diesen Wiki-Link (http://de.wikipedia.org/wiki/Bellsche_Ungleichung#Quantenmechanische_Mittelwert e):


Grüsse, rene

richy
28.07.09, 18:20
Hi Bauhof
"Mainstream" ist tatsaechlich keine gelungene Bezeichnung. Von der Mehrheit akzeptierte, anerkannte Theorien ist damit gemeint.
Wobei die VWT auch akzeptiert ist, aber nicht von der Mehrheit.

dann bedanke ich mich bei dir, dass du mich in einem Atemzug mit Prof. Harald Lesch nennst.
Ich finde Leschs Sendungen interessant und unterhaltsam. Die sind super gemacht. Aber im Falle der nicht Lokalitaet und nicht Realitaet kennt er keine Erklaerung. Seine Privatinterpreation ist mit grosser Sicherheit falsch. Auch die VWT kann nicht alles erklaeren, aber wenigstens einen Teil. Immerhin.
Auf die Ergebnisse von Zeilinger bin ich sehr gespannt.
Er vertritt zwar eine andere Meinung, aber macht das einzigst Richtige. Ueber Experimente festzustellen welche Interpretation denn nun am sinnvollsten ist.
Wobei ich noch immer nicht verstanden habe was er denn wirklich vor hat.
Gruesse

richy
28.07.09, 18:28
Hi rene
Vielen Dank fuer die Erklaerung. Dass der Versuch die Nichtlokalitaet zeigt ist mir schon klar. Probleme hatte ich mit der Nicht Realitaet.
Aber ich meine dies nach deiner Erklaerung etwas besser verstanden zu haben.
Der Ansatzpunkt waere, dass man die Wahrscheinlichkeit beider Ereignisse betrachtet und diese fuerht zu dem Produkt der einzelnen Wahrscheinlichkeiten. Fuer ungleiche Winkel.
Das muss ich jetzt nochmals genauer durchgehen.

Vielen Dank

@uli
Eine Erklaerung im Spin Beispiel koennte sein, dass wir den Gesamtspin "des Universums" gar nicht kennen.
Man weiss nicht welche Werte noch realisiert werden.
Im Prinzip zeigt dies der La Placesche Daemon. Der deshalb scheitert.

rene
28.07.09, 18:55
Hi richy

You’re welcome.

Ganz grob umschreibt der Realitätsbegriff die Unmöglichkeit einer für den direkten Signalsaustausch erforderlichen Überlichtgeschwindigkeit, die ja im Widerspruch zu Einsteins Relativitätstheorie stünde, während mit der Nicht-Lokalität (oder Globalität wie es du so treffend ausdrückst) das verschränkte Quantenpaar unabhängig seiner räumlichen Ausdehnung als globale Entität verstanden werden soll, das nicht mehr unter dem Gesichtspunkt zweier lokal getrennter Teilchen betrachtet werden darf. Somit führt dies zum Widerspruch zwischen der quantenmechanischen Anwendung der Bellschen Ungleichungen mit lokalen und realistischen Theorien.

Grüsse, rene

Gandalf
28.07.09, 21:00
Hi richy!

Schon mein Primitiv Modell leistet beides. Erklaert die Nicht Lokalitaet, Fernwirkung, die Bellsche Ungleichung.
Den Mechanismus der Weltenauswahl (Zusammebruch der Ueberlagerung) erklaert mein Modell nicht.
Und natuerlich keinesfalls den objektiven Zufall.
Das Heim Modell ist eine Anleitung wie man alles erklaeren koennte.
Physikalische und abstrake Welt. Sogar den Urgrund vor dem Urknall.
Aber das Programm des Zufalls kann auch Heim nicht erklaeren.
Die Mechanismen schon.
Warum kann Heim das Programm nicht erklaern ?
Weil dieses das Gesamtprogamm des Universums ist !
Wie kommt diese Programm zustande ?
Weil das Universum in unglaublich feinen Abstaenden sich selbst abtastet und diese Information an G4 weiterleitet.
Ungeheuerliche Vorstellung nicht :-)
Was sind denn die Sensoren fuer diese Messung ?
Jedes Elemetarteilchen !
So ein Quatsch plappert der Kopenhagener der wie selbstverstaendlich die abstrakte Welt mit einbezieht ohne nichteinmal in der Lage ist sein bewusstes Dasein zu quantifizieren.

Soweit alles klar ?

Ich denke Du liegst schon richtig: Die "Standard- und Allerweltserklärungen" ^^ zur QT sind von vorne herein von "Kopenhagener Vorstellungswelten" durchseucht, dass sie allerorten das Zeug dazu haben, nur Verwirrung zu stiften, anstatt aufzuklären. Wenn überhaupt, werden gültige, weil logisch konsinstente Alternativbeschreibungen als "Kritik" (hinten-)angeführt.

Interessant ist auch, dass zur angeführten wiki-Seite zu den "Bellschen Ungleichungen" (http://de.wikipedia.org/wiki/Bellsche_Ungleichung)sich noch eine 'Altversion' im Netz befindet, die vieles viel verständlicher erklärt, als die Aktuelle: http://wiki.rentaweb.ch/index.php/Bellsche_Ungleichung (u.a. auch zu Lokalität und Realismus, "freier Wille", etc.)
Warum die Jetzige in weiten Teilen so ausgedünnt und "verschlimmbessert" wurde, weis ich nicht.

Zum "Mechanismus der Weltenauswahl": Es dürfte damit zusammenhängen, was wir hier schon andiskutiert haben: Mit dem 'Abbruch der Berechnung' (http://www.quanten.de/forum/showpost.php5?p=38955&postcount=204)(nicht der Funktion) .. und mit "Berechnungen" solltest Du eigentlich in Deinem Element sein ;)

Kurzum: Es geht um das 'Halteproblem'. Wie Du weist, bin ich der Ansicht, dass Quantensysteme ihre Umwelt simulieren, um die Autopoiese (http://de.wikipedia.org/wiki/Autopoiesis) aufrecht zu erhalten. Und mit "Simulation der Umwelt" meine ich nicht, die Höhle, in der ein Bär seinen Schutz geniest, oder die Nase mit der er giftiges von Essbarem unterscheidet, oder wenn ein Bakterium "Gift auskotzt", um sich selbst das Feld zu bereiten. - Nein: Es muss immer 'ein ganzes Universum' für das jeweilige System simuliert werden, damit die Quantenberechnung "rund läuft" (die "Höhle", die der Bär "baut" besteht ja aus mineralischen Elementen, die wiederum durch Erdaktivitäten vorgefunden werden können, wobei diese Elemente selbst in einer Sonne oder Supernova... - Wieviele "Gigaterraflops" müssten hier wohl nötig sein?)

Simulationen in einer Endlosschleife, bzw. ohne Ergebnis, wären fatal. Es muss also "jemand" 'die Verantwortung' übernehmen, die Berechnung abzubrechen und auf 'Informationen zu verzichten' (-->Photonen verlassen das System Richtung schwarzes Weltall - Dekohärenz) Und wie bei der Berechnung mit einem klassischen Computer kann das Halteproblem nicht 'innerhalb des (Berechnungs-)Systems slebst gelöst werden. Da aber das Berechnungssystem das 'Universum ist', - kann die Entscheidung über den Abbruch nur von "außerhalb" getroffen werden. Und wenn es jetzt das Multiversum nicht gäbe, - müsste man es wohl erfinden: Damit überhaupt so etwas wie Realität enstehen kann, sind Interferenzen mit anderen Universen unabdingbare Voraussetzungen: Nur dadurch gelingt es Systemen sich "von außen" zu betrachten und Entscheidungen ("Weltenauswahl") zwischen den Welten zu treffen, ohne dass die "Überlagerung tatsächlich zusammenbricht"

Grüße

henning111
09.01.10, 17:14
Und ich bin für fernhafte Spuck-Wirkung ...:D :D :D
Gruß, möbius

... und ich für die Teleportation von Feen ... ...:D :D :D